LSAT and Law School Admissions Forum

Get expert LSAT preparation and law school admissions advice from PowerScore Test Preparation.

 Administrator
PowerScore Staff
  • PowerScore Staff
  • Posts: 8927
  • Joined: Feb 02, 2011
|
#23072
Complete Question Explanation

Method of Reasoning-CE. The correct answer choice is (C)

Anita and Jean are concerned about what is causing the decline in the spotted owl population.

Anita asserts that timber companies cause the decline.

Jean claims that timber companies do not cause the decline, and points out a possible alternate cause: The barred owl might be causing the decline by driving the spotted owl from its habitat.

You should realize that Jean has actually used one of the formulaic methods for weakening a causal argument-- she has offered an alternative cause. The right answer choice will be about that method of reasoning.

Answer choice (A) Jean never denies any of Anita's information; Jean merely contests Anita's conclusion, so this choice is wrong.

Answer choice (B) Since Anita never made any argument involving the survival of the spotted owl species, Jean did not counter any such assumption, and this choice is incorrect. You should not assume that when Anita uses the word "alarming" she has to reference extinction.

Answer choice (C): This is the correct answer choice. Proposing an alternative explanation for an event covers the idea of proposing an alternative cause, and you should realize that "alternative explanation" and "alternative cause" are in this case interchangeable.

Answer choice (D) Since you cannot assume that Jean's "the past three decades" covers more time than Anita's "since 1960," you cannot know whether Jean is using a longer time period, and this choice is unsupported, and wrong.

Answer choice (E) Jean never contested the idea that the spotted owl population is in decline. She merely suggested that there was a different explanation than the one Anita offered. This choice is wrong, because it implies that the spotted owl population might not be in decline, and Jean actually stated that the spotted owl population is in decline.
User avatar
 shanhickey
  • Posts: 21
  • Joined: Apr 11, 2022
|
#94716
Hello!

I found myself trying to decide between A and C, and I understand why C could be correct. But how is A wrong? If Jean is responding to Anita by saying, "No, the decline is not because of the timber companies", is that not denying Anita?
 Rachael Wilkenfeld
PowerScore Staff
  • PowerScore Staff
  • Posts: 1358
  • Joined: Dec 15, 2011
|
#94782
Hi shanhickey,

The key here is to think about what Jean is denying. Arguments are made up of premises (reasons) and conclusions drawn from those premises. We need to know before looking at the answer choices exactly how it is that Jean is countering Anita.

Answer choice (A) says that Jean denies the truth of a premise that the timber companies have been clearing the old growth forests. But we don't see Jean even addressing that premise. We certainly don't see her deny that they have been clearing the forests. Jean's argument moves from the idea that EVEN IF the timber companies are clearing the forests, we can't conclude that is the reason the spotted owl population is declining.

We can also think about where Jean and Anita agree. They both agree the spotted owl population is declining. That's an effect that they both want to explain. Anita suggests it's the timber companies' behavior that is causing the decline. Jean doesn't challenge that the timber companies are cutting down forests, but instead offers another explanation for the effect.

Hope that helps.

Get the most out of your LSAT Prep Plus subscription.

Analyze and track your performance with our Testing and Analytics Package.